Sie sind auf Seite 1von 51

Retina/Vitreous

Question 1 of 130
A 27-year-old woman has had a sudden onset of a dark area inferior to fixation in the left eye earlier in the day. Examination
reveals a visual acuity of 20/20 in each eye and an area of retinal whitening that corresponds to a branch coming from the
superotemporal retinal artery. Which of the following is least likely to be associated with this condition?

An abnormality demonstrated with carotid Doppler and/or ultrasound imaging techniques

Evidence of intravenous drug use

A history of migraine

An abnormality detected on cardiac echography


Please select an answer
Feedback: Branch retinal artery occlusion is an unusual occurrence in young people, and the causes are, in general, quite
different from such occlusions in the elderly. Cardiac valve abnormalities and cardiac myxoma can lead to embolic
phenomena, as can oral contraceptives. Exogenous emboli (talc) are seen in intravenous drug abusers. Migraine can cause
true infarction and is the most common cause in someone of this age. Carotid disease is almost never involved in arterial
occlusions in persons under 40 years of age.

Question 2 of 130
In which of the following quadrants are retinal dialyses most often found following blunt trauma?

Inferotemporal, superonasal

Superonasal, inferonasal

Superotemporal, superonasal

Inferotemporal, superotemporal
Please select an answer
Feedback: Blunt trauma may result in a variety of retinal breaks including retinal dialyses, macular holes, giant retinal tears,
horseshoe retinal tears, operculated retinal tears, and large, necrotic, sometimes posterior retinal breaks often associated
with chorioretinitis sclopetaria. Retinal dialyses, however, are the most common retinal breaks found following blunt ocular
trauma. Retinal dialyses resulting from blunt trauma are most often found in the inferotemporal and superonasal quadrants.
All patients with a history of blunt trauma need to be given a careful retinal examination with scleral depression to look for
retinal breaks after their pain, iritis, or hyphema, if any, has resolved.
Question 3 of 130
Which of the following statements is least accurate regarding acquired color vision defects?

Acquired color vision defects may result from macular or optic nerve disease.

The Farnsworth-Munsell 100-hue test may be used to detect discrimination loss in patients with acquired color vision
defects.

Acquired color vision defects typically resemble tritan defects.

Pseudoisochromatic plates may not be reliably used to detect acquired color vision defects.
Please select an answer
Feedback: Acquired color vision defects may occur as the result of optic nerve or retinal disease that may or may not be
hereditary. Acquired color vision defects are variable, producing mild to severe color defects of variable types. Fluid in or
under the macula more often affects the blue-yellow (tritan) axis than the red-green axis. Degenerative retinal conditions
such as Stargardt's disease often produce a red-green defect. Optic nerve disease, such as Leber's hereditary optic atrophy,
tends to affect the red-green axis more than the blue-yellow axis. Pseudoisochromatic plates are helpful in screening for X-
linked color vision defects, but are not reliable in detecting acquired color defects. Sensitive tests of color discrimination,
such as the Farnsworth-Munselll 100-hue test, may be the best for detecting variable and sometimes mild acquired color
defects.

Question 4 of 130
According to the Early Treatment Diabetic Retinopathy Study (ETDRS), which of the following clinical findings is not used to
determine if a patient has very severe nonproliferative (preproliferative) diabetic retinopathy?

Severe retinal hemorrhages in all four quadrants of the fundus

Cotton-wool patches in all four quadrants

Venous beading in at least two quadrants

Moderately severe intraretinal microvascular abnormalities in at least one quadrant


Please select an answer
Feedback: The ETDRS examined early panretinal photocoagulation in eyes with severe nonproliferative changes. Five-year
rates of severe visual loss were found to be similar between groups that underwent treatment, and those for whom treatment
was deferred. However, due to side effects, such as small decreases in visual acuity and peripheral visual field,
recommendations were made to defer panretinal photocoagulation until retinopathy was approaching a high-risk stage. Fifty
percent of eyes that were approaching the high-risk stage were found to reach high-risk proliferative retinopathy within 12 to
18 months. Factors used to determine whether a patient was approaching high-risk retinopathy included severe retinal
hemorrhages in all four quadrants of the fundus, venous beading in at least two quadrants, and moderately severe
intraretinal microvascular abnormalities in at least one quadrant. Although cotton-wool spots are frequently visible in eyes
with capillary nonperfusion in proliferative diabetic retinopathy, the ETDRS concluded that these were not as predictive in
determining which eyes were approaching high-risk retinopathy.
Question 5 of 130
Which of the following statements about patients who inherit sickle cell hemoglobin is least accurate?

Those with hemoglobin C and Sickle thalassemia have the most serious ocular complications.

Those with hemoglobin SS have the worst systemic complications.

Sickling of red cells occurs under conditions of decreased oxygen tension.

"Salmon-patch" lesions, "black-sunburst" lesions, and "sea fans" are all signs of proliferative sickle cell retinopathy.
Please select an answer
Feedback: Approximately 10% of North American blacks of Central and West African origin have abnormal hemoglobin:
approximately 8% to 9% have AS hemoglobin, 0.4%, SS disease, 0.1% to 3% SC disease, and 0.5% to 1.0% S
thalassemia. Salmon-patch and iridescent spot lesions are due to intraretinal hemorrhages, while black-sunburst lesions are
due to subretinal hemorrhage and arise from hyperplasia of the retinal pigment epithelium. Sea fans are fronds of
neovascularization that extend from the retina into the vitreous and are therefore signs of proliferative retinopathy. A
prospective clinical trial has demonstrated the efficacy of argon laser scatter photocoagulation therapy for proliferative sickle
cell retinopathy. Prolonged loss of visual acuity and vitreous hemorrhage were reduced in treated eyes compared with
controls. Scatter photocoagulation proved to be effective and safe in the treatment of patients with sea-fan
neovascularization.

Question 6 of 130
Within several hours of an accident, you see a 23-year-old farmer who injured his right eye while hammering on a bearing.
His visual acuity is 20/200 OD and 20/20 OS, and he has a definite relative afferent pupillary defect. His anterior segment is
clear, except for some cells and flare in the anterior chamber. A vitreous haze obscures fine details of the posterior pole;
however, you see sheathing of some of the peripheral retinal vessels. The management of endophthalmitis in this situation
should not include which of the following strategies?

Lensectomy and vitrectomy, followed by intravitreal injection of broad-spectrum antibiotics

Echography and/or CT scan for identification and localization of an intraocular foreign body

Culture and sensitivities of the intraocular foreign body

Intravitreal injection of antibiotics to cover gram-positive and gram-negative organisms


Please select an answer
Feedback: The management of endophthalmitis is controversial. It is prudent under the circumstances to perform a
vitrectomy and remove the intraocular foreign body under direct visualization. The foreign body should be submitted for
culture and sensitivities. At the conclusion of the procedure, intravitreal and subconjunctival antibiotics covering both gram-
positive and gram-negative organisms should be administered. This protocol is especially important in a rural setting, where
more than one type of organism may have entered the eye. Vitrectomy is probably useful in the type of endophthalmitis
described, but it is questionable whether lensectomy is warranted at this stage.
Question 7 of 130
During phacoemulsification for cataract, one-third of the lens nucleus drops into the posterior vitreous of a 76-year-old
patient. Which of the following statements would not be suggested for the treatment of this patient?

The nuclear material need not be surgically removed immediately; resultant inflammation or increased intraocular
pressure may be managed medically.

The cataract surgeon should attempt to irrigate the lens fragments from the anterior vitreous if they are visible.

Removal of the lens fragment by a vitreoretinal surgeon may be indicated, depending on its size, the resultant
inflammation, and intraocular pressure control.

Placement of an intraocular lens may still be considered in this patient.


Please select an answer
Feedback: Retained unencapsulated lens material can produce severe, granulomatous intraocular inflammation, and may
lead to secondary glaucoma, corneal decompensation, and retinal detachment. Depending on the size of the retained
nuclear material, the degree of resultant inflammation, and increased intraocular pressure, surgical removal of the retained
lens by posterior vitrectomy may or may not be mandatory. Intensive topical corticosteroids and pressure-lowering
medications may control the secondary inflammation and glaucoma induced by small nuclear fragments. However, larger
nuclear fragments may produce protracted inflammation and glaucoma, necessitating removal by posterior vitrectomy. If the
lens nucleus is dropped posteriorly during cataract extraction, the surgeon should clean any vitreous that may have come
forward through the ruptured lens capsule, consider placing the posterior chamber intraocular lens if capsular support is
adequate, and close the cataract wound in a watertight fashion. However, removal of lens material from the anterior vitreous
by vigorous irrigation should not be attempted, because it may produce vitreoretinal traction, retinal breaks, and retinal
detachment. The patient can be observed and treated medically if the nuclear fragment is small, but should be referred
promptly to a vitreoretinal surgeon postoperatively if a larger nuclear fragment is present.

Question 8 of 130
Which type of hemoglobinopathy puts a patient most at risk for developing proliferative retinopathy?

SS

AC

AS

SC
Please select an answer
Feedback: Normal hemoglobin is referred to as "A," with amino acid substitutions producing either "S" (sickle hemoglobin)
or "C," and inadequate globin chain synthesis producing "Thal" (thalassemia). Patients homozygous for S hemoglobin have
sickle-cell anemia and manifest severe systemic complications from this hemoglobinopathy; rarely, however, do they have
proliferative retinopathy. Patients who are heterozygous for one of the sickle hemoglobins (AS or AC) generally have a more
benign systemic course and also occasionally, but rarely, have retinopathy. Patients who are heterozygous for two abnormal
hemoglobins (SC or SThal) tend to have mild anemia and a benign systemic course compared to patients with sickle-cell
disease; however, they are most likely to develop proliferative retinopathy.
Question 10 of 130
Which of the following statements does not accurately describe routine evaluation of patients with diabetes mellitus?

Pregnant diabetic women should be examined in the first trimester and, at a minimum, every 3 months thereafter.

Newly diagnosed diabetics under the age of 30 should have their first ophthalmologic examination within 5 years of the
onset of their disease.

Newly diagnosed diabetics 31 years of age or older should have their first ophthalmologic examination at the time of
diagnosis.

Routine follow-up of a diabetic under the age of 30 should take place every 2 years at a minimum, if there has been no
evidence of diabetic retinopathy to date.
Please select an answer
Feedback: Studies have demonstrated a relationship between the prevalence and severity of diabetic retinopathy and the
duration of diabetes mellitus. In patients diagnosed with diabetes up to the age of 30, diabetic retinopathy was detected in
18% of patients having the disease for 3-4 years, but increased to 80% in patients who had been diabetic for 15 years or
more. Significant retinopathy, however, rarely develops prior to puberty; and in patients between 10 and 30 years of age, it
usually is not seen until the patient has diabetes for 6-7 years. Therefore, most young-onset diabetics who develop
significant retinopathy will have the retinopathy discovered if their initial ophthalmic examination is within 5 years of their
being diagnosed with diabetes. In diabetics diagnosed after age 30, significant retinopathy has been found in 3% of patients
at the time of diagnosis; therefore, an initial examination should be performed on diagnosis of the diabetes mellitus and
annually thereafter. Diabetic retinopathy can progress rapidly during pregnancy; therefore, pregnant diabetic women need to
be followed carefully, with examinations performed in the first trimester and, at a minimum, every 3 months until delivery.

Question 11 of 130
Which of the following is not an indication for immediate or early pars plana vitrectomy in a penetrating injury?

Vitreous incarceration in a posterior scleral wound that cannot be closed by an external approach

Retinal detachment with vitreous hemorrhage

Early endophthalmitis

Intraocular foreign body


Please select an answer
Feedback: Immediate or early pars plana vitrectomy is indicated when an intraocular foreign body is present to decrease
the risk of endophthalmitis, or to reduce the inflammatory effects that the intraocular foreign body may have on ocular
tissues. Traumatic endophthalmitis, particularly that following an intraocular foreign body, may be associated with
aggressive organisms such as Bacillus cereus. Although the benefit of pars plana vitrectomy in endophthalmitis is a matter
of debate, the procedure is advisable when potentially aggressive organisms are likely to be present. Early (not necessarily
immediate) pars plana vitrectomy should be performed to repair retinal detachment when vitreous hemorrhage prevents
adequate visualization to allow scleral buckling. Vitreous incarceration in a posterior scleral wound also requires pars plana
vitrectomy; however, when an unclosed posterior wound is present, pars plana vitrectomy should be delayed 7 days to allow
possible spontaneous closure of the wound prior to proceeding with vitrectomy.
Question 12 of 130
Which of the following statements is not accurate regarding retinal detachment patient selection for pneumatic retinopexy?

Retinal break(s) should be located within the superior two-thirds of the fundus.

Aphakic or pseudophakic patients have the same rate of success with pneumatic retinopexy as phakic patients.

Patients with proliferative vitreoretinopathy (PVR) grade C or higher or severe glaucoma should not be considered
candidates for pneumatic retinopexy.

Retinal break(s) should be located within 1 clock hour of each other.


Please select an answer
Feedback: Published studies demonstrate that pneumatic retinopexy for retinal detachment may have a degree of success
comparable to scleral buckling in selected cases. Most studies limited patient selection to those retinal detachments that had
retinal breaks within the superior two-thirds of the fundus, and all were located within 1 clock hour of each other. Patients
with PVR grade C or higher, in which the retina may be stiff or the retinal breaks may be under traction, are considered poor
candidates for pneumatic retinopexy. Patients with a history of severe glaucoma may also be poor candidates, because of
the significant rise in intraocular pressure that can occur immediately following gas injection. Although failure rates are
reportedly higher in pseudophakic or aphakic patients (compared to phakic patients) following either pneumatic retinopexy or
scleral buckling, pseudophakia or aphakia is generally not considered an absolute contraindication to pneumatic retinopexy

Question 13 of 130
In the evaluation and treatment of diabetic retinopathy, what is fluorescein angiography least useful for?

Determining the location of retinal microaneurysms prior to photocoagulation

Determining the presence of clinically significant diabetic macular edema prior to recommending photocoagulation

Determining areas of capillary closure

Determining the location of diffuse retinal leakage from incompetent retinal capillaries or intraretinal microvascular
abnormalities prior to photocoagulation
Please select an answer
Feedback: The Early Treatment Diabetic Retinopathy Study defined clinically significant diabetic macular edema by
biomicroscopic examination of the macula, and not by fluorescein angiography. Therefore, recommendations for
photocoagulation should be based on biomicroscopic determination of: (1) retinal thickening at or within 500 microns of the
center of the fovea; (2) hard exudate at or within 500 microns of the center of the fovea, if associated with thickening of
adjacent retina; or (3) a zone or zones of retinal thickening 1 disc area or larger, any part of which is within 1 disc diameter
of the center of the fovea. Fluorescein angiography may be useful in evaluating the presence and location of capillary
closure, particularly when there is clinically unexplainable visual loss. Fluorescein angiography is also useful prior to
photocoagulation to determine the location of leaking retinal microaneurysms or areas of diffuse retinal leakage that require
treatment.
Question 14 of 130
A vitreous biopsy of a 66-year-old patient demonstrates cryptococcal organisms on fungal stain and culture. Which of the
following statements does not accurately describe endogenous ocular cryptococcal infections?

Cryptococcal infections occur only in immunosuppressed patients.

Chorioretinitis is the most common intraocular presentation of Cryptococcus.

Ocular involvement frequently is the result of direct extension along the optic nerve or by hematogenous spread.

Cryptococcal meningitis is frequently associated with ocular cryptococcal infections.


Please select an answer
Feedback: Cryptococcal infections are caused by Cryptococcus neoformans, an ubiquitous fungus found in bird droppings.
Cryptococcal infections are more common in, but not limited to, immunocompromised patients. Chorioretinitis, with or
without vitritis, is the most common intraocular presentation of cryptococcosis; cryptococcal endophthalmitis is rare. Many
disorders must be considered in the differential diagnosis of chorioretinal lesions with vitritis, such as those seen in
cryptococcosis. Similar lesions may be found in tuberculosis, sarcoidosis, CMV retinitis, candidiasis, and toxoplasmosis.
Acute retinal necrosis syndrome also presents with retinal inflammation and vitritis, but given this patient's age, would be
unlikely. Large-cell lymphoma should also be considered in the differential diagnosis. Ocular cryptococcal organisms may
gain access to the eye by hematogenous spread, by direct inoculation as in trauma, or by direct extension along the optic
nerve from the often accompanying cryptococcal meningitis, which is often fatal.

Question 15 of 130
A 60-year-old man presents with a 3-day history of photopsias and new floaters in his left eye. His visual acuity is 20/20 OU.
Slit-lamp examination shows mild nuclear sclerosis and clear anterior vitreous bilaterally. On fundus examination, a posterior
vitreous detachment (PVD) is seen only in the left eye. Which of the following statements best describes this situation?

Myopia, diabetic retinopathy, vitreous hemorrhage, and surgical aphakia all predispose the patient to vitreous
detachment at an earlier age.

If hemorrhage or pigment granules are not present in the vitreous, depressed examination of the peripheral retina is
not necessary.

Approximately 15% of patients who present with acute, symptomatic PVD will have a retinal tear.

In the majority of cases, when the fellow eye develops a PVD it will likely respond in the same way (ie, symptoms,
complications) as the first eye did upon developing PVD.
Please select an answer
Feedback: Symptomatic PVD is characterized by photopsias and/or floaters, symptoms that may also occur with the
development of a retinal tear. Since approximately 15% of patients presenting with an acute, symptomatic PVD will have a
retinal tear, all such patients should have a dilated examination of the peripheral retina with indirect ophthalmoscopy and
scleral depression to look for peripheral retinal breaks. The presence of hemorrhage or pigment (Shafer's sign) in the
vitreous correlates highly with the presence of a retinal break. However, absence of Shafer's sign does not negate the need
for careful examination of the peripheral retina. Posterior vitreous detachment is more common with increasing age, but
myopia, vitreous hemorrhage, surgical aphakia, and diabetic retinopathy, particularly following panretinal photocoagulation,
all have been associated with the development of posterior vitreous detachment at an earlier age. Fellow eyes of patients
who have developed a PVD in one eye most often follow a similar course (symptoms, complications) when a PVD develops.
Question 16 of 130
Which of the following statements does not accurately describe central retinal artery occlusion?

Electroretinography shows a diminished A-wave.

Iris neovascularization may occur in 5% of patients.

Emboli more commonly cause retinal arterial occlusions than thrombosis or vascular narrowing from atherosclerosis.

Long-term survival is decreased in patients who have had retinal artery occlusions.
Please select an answer
Feedback: Retinal arterial occlusions may occur from many causes including embolism, thrombosis, or narrowing from
atherosclerosis, vasculitis, arterial spasm as in migraine, and extravascular compression. Of the many potential causes,
emboli originating from the carotid arteries are the most common cause of retinal artery occlusion. Multiple reports have
noted decreased life expectancy for patients with retinal artery obstructions. Since the central retinal artery supplies the
inner retina, the B-wave is diminished on the electroretinogram. Iris neovascularization and neovascular glaucoma occur
infrequently (approximately 5%) following central retinal artery occlusion. The development of iris neovascularization may
not always be the direct result of the retinal artery occlusion, since significant carotid artery narrowing has frequently been
noted on the ipsilateral side.

Question 17 of 130
A 32-year-old man presents without complaints for routine examination. On indirect ophthalmoscopy, multiple patches of
peripheral lattice degeneration containing multiple atrophic retinal holes are noted in the superior retina OD. Lattice
degeneration without retinal breaks is noted inferiorly OS. Which of the following statements regarding prophylactic
treatment is most correct?

Both of the patient's eyes should be prophylactically treated with laser photocoagulation or cryoretinopexy.

Only the patient's right eye should be prophylactically treated with laser photocoagulation or cryoretinopexy.

If there is a prior history of retinal detachment in the left eye, prophylactic laser photocoagulation or cryoretinopexy
should be considered in the right eye.

The patient's right eye should receive prophylactic treatment with laser photocoagulation or cryoretinopexy prior to
cataract surgery.
Please select an answer
Feedback: Lattice degeneration occurs in 6% to 8% of the population, with 20% to 30% of patients with lattice degeneration
also having coexisting retinal holes. The decision to treat prophylactically is based on the risk of developing a retinal
detachment without treatment, how much the treatment will reduce the risk of retinal detachment, and the risks of treatment.
The rate of retinal detachment in eyes with lattice degeneration has been estimated to be less than 1%. Retinal breaks
following cataract surgery are less likely to be associated with lattice degeneration than with the development of new flap
tears. There are no studies documenting the value of prophylactic treatment in eyes with lattice degeneration except in the
fellow phakic eyes with lattice degeneration of previous retinal detachment patients. In untreated eyes, a 2.5 times greater
risk (1.8% vs 5%) of a new tear or detachment was found than in treated eyes.
Question 18 of 130
A 24-year-old man with a 10-year history of insulin-dependent diabetes mellitus presents with a visual acuity of 20/25 OD
and 20/200 OS. Examination of the macula in the right eye demonstrates hard exudates and retinal thickening within 500
microns of the foveal center. A small area of flat retinal neovascularization is present in the right eye off the superotemporal
arcade. The vitreous in the right eye is clear. Examination of the left eye demonstrates diffuse retinal thickening throughout
the macula, scattered hard exudates, and blot hemorrhages. Marked neovascularization of the disc is present in the left eye,
as well as nasal retinal neovascularization with mild vitreous hemorrhage. What is the best sequence of photocoagulation
treatment for this patient?

Initial panretinal and focal photocoagulation OS, followed by focal photocoagulation OD

Initial focal photocoagulation OU, followed by panretinal photocoagulation OU

Initial panretinal photocoagulation OS, followed by focal photocoagulation OU

Initial panretinal photocoagulation OU, followed by focal photocoagulation OS


Please select an answer
Feedback: According to the criteria of the Diabetic Retinopathy Study (DRS) and the Early Treatment Diabetic Retinopathy
Study (ETDRS), this patient has both high-risk proliferative diabetic retinopathy (PDR) in the left eye, and clinically
significant diabetic macular edema in both eyes. Therefore, panretinal photocoagulation in the left eye, and focal
photocoagulation in both eyes are needed. However, panretinal photocoagulation has been shown to exacerbate coexisting
diabetic macular edema. The ETDRS found that to reduce the effects of panretinal photocoagulation on diabetic macular
edema, the procedure is best delayed until focal photocoagulation is completed. However, in cases in which high-risk PDR
characteristics and diabetic macular edema coexist, the ETDRS does not recommend delaying panretinal photocoagulation
in lieu of focal photocoagulation. Therefore, in this patient, concurrent focal and panretinal photocoagulation would be the
best choice to initiate treatment for the high-risk PDR in the left eye, while reducing the effects of panretinal
photocoagulation on the diabetic macular edema.

Question 19 of 130
The Diabetes Control and Complications Trial was a multicenter, randomized clinical trial in which insulin-dependent diabetic
patients with either no retinopathy, or mild to moderate nonproliferative retinopathy, were treated either with conventional
insulin therapy, or intensive insulin therapy, that consisted of either three or more insulin injections daily, or an insulin pump.
Which of the following statements does not accurately describe the findings of this trial?

No transient early worsening of retinopathy with intensive insulin therapy was noted in patients with mild to moderate
retinopathy, as reported in previous trials.

In patients with mild to moderate nonproliferative retinopathy, intensive therapy reduced the development of
proliferative or severe nonproliferative retinopathy by 47%, compared to conventional insulin therapy.

In patients initially without retinopathy, intensive insulin therapy reduced the risk of onset of retinopathy by 76%
compared to patients in the conventional therapy group.

In patients initially with mild to moderate nonproliferative retinopathy, intensive insulin therapy slowed the progression
of retinopathy by 54%, compared to patients in the conventional therapy group.
Please select an answer
Feedback: The Diabetes Control and Complications Trial demonstrated that intensive insulin therapy that maintained serum
glucose at much tighter, lower levels than conventional therapy: (1) reduced the risk of development of retinopathy in
patients without retinopathy by 76%; (2) reduced the risk of development of proliferative or severe nonproliferative
retinopathy in patients with mild to moderate nonproliferative retinopathy by 47%; and (3) slowed the progression of
retinopathy by 54% in patients with mild to moderate nonproliferative retinopathy. With intensive therapy, however, patients
with mild to moderate retinopathy often experienced a transient worsening of their retinopathy over the first year of
treatment. Ultimately, these patients demonstrated a significant reduction in the risk of progression of their retinopathy, the
development of proliferative or severe nonproliferative retinopathy, or the need for photocoagulation, as compared to
patients treated with conventional therapy.

Question 20 of 130
Which of the following statements about peripheral uveitis is least likely?

The most common causes of reduction in visual acuity in peripheral uveitis are cystoid macular edema and vitreous
debris.

Fluorescein angiography may demonstrate staining of peripheral retinal vessels.

An indication for initiating treatment for peripheral uveitis is a reduction in visual acuity to less than 20/25.

Lyme disease, Fuchs' heterochromic cyclitis, Toxocara canis, toxoplasmosis, and retinoblastoma are included in the
differential diagnosis of peripheral uveitis.
Please select an answer
Feedback: Peripheral uveitis (formerly called pars planitis) is a panuveitis that most commonly occurs bilaterally in patients
under age 30. Symptoms most frequently include floaters and painless decrease in vision. Characteristic ocular findings
include pars planaora serrata exudate (snowbank), vitreous debris, posterior subcapsular cataract, cystoid macular edema,
optic disc edema, and retinal vasculitis. Less commonly, retinal neovascularization, peripheral angiomalike lesions, and
tractionallrhegmatogenous retinal detachment may develop. Decrease in visual acuity most commonly occurs as the result
of either vitreous debris or cystoid macular edema. Fluorescein angiography may demonstrate cystoid macular edema, optic
nerve papillitis, and retinal vascular staining. There are no diagnostic tests specific for peripheral uveitis; diagnosis is based
on the clinical appearance and the exclusion of other diagnoses. Other ocular inflammatory conditions involving the anterior
and posterior segments that present without pain or injection should be considered, including Lyme disease, Fuchs'
heterochromic cyclitis, Toxocara canis, juvenile rheumatoid arthritis, and toxoplasmosis. In younger children, retinoblastoma
must also be considered in the differential diagnosis. Initial therapeutic intervention with corticosteroids is started if visual
acuity decreases below 20/40, but earlier therapy may be considered in certain clinical situations.

Question 21 of 130
Which of the following constitutes a pertinent clinical feature of neovascular AMD?

Retinal pigment epithelium (RPE) pigmentary changes

Drusen

Subretinal fluid

Geographic RPE atrophy


Please select an answer
Feedback: Neovascular AMD is characterized by the development of choroidal neovascularization. This exudative
neovascular complex commonly results in the accumulation of subretinal fluid. Drusen, retinal pigment epithelium (RPE)
pigmentary changes, and geographic RPE atrophy are characteristics of non-neovascular AMD.
Question 22 of 130
Which disorder is most associated with central retinal venous occlusion?

Acute iridocyclitis

Cataract

Age-related macular degeneration

Primary open-angle glaucoma


Please select an answer
Feedback: The ophthalmic disorder that is most associated with central retinal venous occlusion is primary open-angle
glaucoma. There is no known association between central retinal venous occlusion and age-related macular degeneration,
cataract, or acute iridocyclitis.

Question 23 of 130
Epiretinal membrane is most commonly associated with which one of the following conditions?

Retinal venous occlusion

Uveitis

Trauma

Posterior vitreous detachment


Please select an answer
Feedback: Although epiretinal membrane may be associated with trauma, retinal venous occlusion, and uveitis, it is most
commonly associated with posterior vitreous detachment.

Question 24 of 130
What are the characteristics of a stage-3 macular hole?

Deep foveal yellow spot or ring

Full-thickness hole less than 400 microns in diameter

Full-thickness hole greater than 400 microns in diameter, with or without a cuff of subretinal fluid with a total posterior
vitreous detachment (PVD), with or without an operculum

Full-thickness hole greater than 400 microns in diameter, with or without a cuff of subretinal fluid
Please select an answer
Feedback: A stage-3 macular hole is a full-thickness hole greater than 400 microns in diameter, with or without a cuff of
subretinal fluid. A stage-1 hole is characterized by a deep foveal yellow spot or ring. A stage-2 hole is a full-thickness hole
less than 400 microns in diameter. A stage-4 hole is a full-thickness hole greater than 400 microns in diameter, with or
without a cuff of subretinal fluid with a total posterior vitreous detachment (PVD), with or without an operculum.
Question 25 of 130
Which one of the following represents the most significant risk factor for the development of diabetic retinopathy?

Recent diagnosis of diabetes mellitus

Systemic hypertension

Hyperlipidemia

Poor glycemic control


Please select an answer
Feedback: Of the risk factors, including the recent diagnosis of diabetes mellitus, systemic hypertension, poor glycemic
control, and hyperlipidemia, the most significant one for the development of diabetic retinopathy is poor glycemic control.

Question 26 of 130
Which one of the following epidemiologic factors is most closely associated with central retinal arteriolar occlusion?

Male gender

Fifth decade of life

Pulmonary fibrosis

Thrombocytopenia
Please select an answer
Feedback: Male gender is an epidemiologic factor closely associated with central retinal arteriolar occlusion. This condition
occurs most commonly in the seventh decade of life. There is no known association between central retinal arteriolar
occlusion, and thrombocytopenia or pulmonary fibrosis.

Question 27 of 130
Which of the following is a risk factor for idiopathic central serous chorioretinopathy?

Anticoagulant use

Coexisting cancer

Pregnancy

Female gender
Please select an answer
Feedback: Idiopathic central serous chorioretinopathy has been associated with pregnancy. Additional associations include
lupus, organ transplantation, hemodialysis, and corticosteroid use. The disorder is more common in men. There is no known
association with coexisting cancer or anticoagulant use.
Question 28 of 130
What is the most common etiology of the vitreomacular traction syndrome?

Inflammatory disease

Idiopathic

Metabolic disease

Vascular occlusion
Please select an answer
Feedback: The vitreomacular traction syndrome represents persistent vitreous macular and/or peripapillary traction in an
eye with a partial posterior vitreous detachment. This condition is most commonly idiopathic. It may also be associated with
inflammatory diseases, vascular occlusions, and metabolic diseases. It is most prevalent in patients more than 60 years old.

Question 29 of 130
Branch retinal venous occlusion is most commonly associated with which systemic disease?

Osteoarthritis

Hypothyroidism

Diabetes insipidus

Arterial hypertension
Please select an answer
Feedback: Risk factors for branch retinal venous occlusion include diabetes mellitus, hypertension, hyperlipidemia, and
primary open angle glaucoma. Branch retinal venous occlusion is most commonly associated with systemic hypertension.

Question 30 of 130
Pattern macular dystrophies typically present in which period of life?

3rd decade

2nd decade

1st decade

4th decade
Please select an answer
Feedback: Pattern macular dystrophies typically present in the 4th to 6th decade of life. This, along with their commonly
autosomal dominant inheritance, differentiates these disorders from other macular dystrophies and degenerations.
Question 31 of 130
Which one of the following patient characteristics is associated with the increased quality of fluorescein angiographic
images?

Normal renal function

Normal intraocular pressure

Clear ocular media

Intraocular gas bubble


Please select an answer
Feedback: Clear ocular media (and a skilled photographer) is very helpful in obtaining high quality fluorescein angiographic
images.

Question 32 of 130
Which one of the following is included in first-line therapy for macular-threatening toxoplasmosis chorioretinitis?

Oral antibiotics

Topical cycloplegics

Topical antibiotics

Intravenous antibiotics
Please select an answer
Feedback: First-line therapy for macular-threatening toxoplasmosis chorioretinitis includes oral antibiotics, including
sulfadiazine, primethamine, and folinic acid. Peripheral lesions may be treated with clindamycin or Bactrim. Oral steroids
may be used as an adjuct, but only 24 hours after starting antibiotics. Topical antibiotics and IV antibiotics have no role in
toxoplasmosis therapy. Topical cycloplegics are typically not necessary.

Question 33 of 130
What is the presumed cause of venous obstruction in branch retinal vein occlusion?

Veins develop sclerosis after occlusion

The retinal artery and vein share a common adventitial sheath

Embolism

Retinal hemorrhages result in reduced venous flow


Please select an answer
Feedback: Systemic hypertension causes hardening of the retinal arterioles. At the point where the retinal arteriole and
venule cross, they are connected by a common adventitial sheath, which binds them together. The sclerotic arteriole causes
a deformation of the venules lumen, resulting in turbulent blood flow and a blood clot.
Question 34 of 130
Which one of the following is traditionally a contraindication to fluorescein angiography?

Prior allergic reaction to shellfish

Difficult venous access

Prior allergic reaction to iodine

Pregnancy
Please select an answer
Feedback: There is no evidence that intravenous administration of flurescein has an adverse effect on the developing fetus,
but traditionally fluorescein angiography is avoided in pregnant women.

Question 35 of 130
When performing cataract extraction surgery, what are the signs of expulsive choroidal hemorrhage?

Hypopyon

Wound gape, vitreous presentation, and iris prolapse

Collapse of the globe

Floppy iris
Please select an answer
Feedback: Expulsive choroidal hemorrhage is one of the most devastating acute complications of intraocular surgery. It is
caused by prolonged ocular hypotension, which causes rupture of a choroidal vessel, and allows suprachoroidal
hemorrhage to go unchecked. As the hemorrhage expands quickly, the intraocular contents may begin to extrude, resulting
in wound gape, vitreous presentation, and iris prolapse.

Question 36 of 130
Which of the following interventions is the most appropriate management following fluorescein angiography?

Administer aspirin

Perform urine culture and sensitivity for urine color change

Observe patient for late adverse reactions

Avoid sunlight for 5 days following the procedure


Please select an answer
Feedback: Allerigic reactions to fluorescein dye may occur immediately after the administration of the dye, or several hours
later. Adverse effects of the dye are usually mild, but rarely can be serious or even fatal.
Question 37 of 130
When evaluating a choroidal nevus, a clinical feature that would not increase suspicion for malignant transformation would
be which of the following?

Presence of subretinal fluid

Presence of orange pigmentation

Thickness greater than 3mm

Presence of drusen
Please select an answer
Feedback: When evaluating a choroidal nevus, the presence of subretinal fluid, hemorrhage, orange pigmentation, and
thickness greater than 3 mm should increase suspicion for malignant transformation to choroidal melanoma. Presence of
drusen on the lesion is a reassuring sign of chronicity.

Question 38 of 130
A 42-year-old woman from Mexico, who has two pet cats, presents with a yellow-white lesion in the macula with overlying
vitritis. Adjacent to the lesion is a pigmented retinal scar. Visual acuity in the affected eye is 20/80. What is the most
appropriate intervention?

Combined therapy with pyrimethamine, sulfadiazine, leucovorin, and oral steroids

Monotherapy with doxycycline

Oral steroids alone

Monotherapy with leucovorin


Please select an answer
Feedback: Toxoplasmosis chorioretinitis is commonly treated with a combination of pyrimethamine, sulfadiazine, and
leucovorin, followed by oral steroids. Doxycycline is sometimes used when the patient is not able to tolerate first-line
therapy.
Question 39 of 130
Which of the following are defining characteristics of severe nonproliferative diabetic retinopathy?

Vitreous hemorrhage with a subhyaloid hemorrhagic component

Tractional retinal detachment

Intraretinal hemorrhages, venous beading, and intraretinal microvascular abnormalities

Early retinal neovascularization


Please select an answer
Feedback: Intraretinal hemorrhages, venous beading, and intraretinal microvascular abnormalities are typical features of
severe nonproliferative diabetic retinopathy. Retinal neovascularization, vitreous hemorrhage, and tractional retinal
detachment are features of proliferative diabetic retinopathy.

Question 40 of 130
Which of the following conditions is typically associated with dense vitritis?

Punctate inner choroiditis

Acute retinal necrosis

Sarcoidosis

Toxoplasmosis
Please select an answer
Feedback: Toxoplasmosis typically presents as vascular sheathing and retinal necrosis, often adjacent to an old scar, with a
dense vitritis. Sometimes the vitritis is so dense that is resembles a "headlight in the fog". Sarcoidosis and PIC typically have
little, if any, vitritis. ARN may present with vitritis, but it is typically not as dense as that which accompanies toxoplasmosis.

Question 41 of 130
Late onset complications (> 6 weeks) of scleral buckle procedures includes which of the following?

Retinal incarceration

Buckle extrusion

Choroidal neovascularization

Choroidal detachment
Please select an answer
Feedback: Buckle extrusion is an uncommon complication of scleral buckle procedure. It may be accompanied by pain,
bleeding, and infection. Treatment is to remove or modify the buckle. Retinal incarceration and choroidal detachment are
early complications of scleral buckle procedure. Choroidal neovascularization is not a complication of scleral buckle
procedure.
Question 42 of 130
The technique of fluorescein angiography includes which of the following procedures?

Intravenous injection of 5ml of ICG solution

Intramuscular injection of 5 ml of 10% sodium fluorescein

Intravenous injection of 5 ml of 10% sodium fluorescein

Coadministration of oral or IV benadryl


Please select an answer
Feedback: Fluorescein angiography involves intravenous injection (or less commonly oral administration), or fluorescein
dye, followed by photography of the retinal vasculature. Intramuscular administraton of fluorescein is never appropriate. ICG
is used intravenously in ICG angiography. Benadryl may be used if an allergic reaction to fluorescein is suspected.

Question 43 of 130
On routine ophthalmoscopic examination, a 62-year-old woman is noted to have asymptomatic, bilateral, smooth peripheral
elevations in the inferotemporal retina that extend slightly posterior to the equator. Visual acuity is 20/20 OU. Which of the
following statements best relates to this situation?

Laser demarcation along the posterior border of these lesions will prevent extension into the macula.

Retinal detachment frequently occurs in such a case if an inner layer retinal break is present.

Retinal detachments associated with outer-layer breaks typically progress rapidly.

Retinal detachment will not occur unless a retinal break is present in the outer layer or full thickness retina.
Please select an answer
Feedback: Senile, or acquired, retinoschisis is present in up to 4% of normal patients over age 40. This frequently bilateral
condition typically develops from a splitting of the outer plexiform layer in the peripheral retina. Smooth, tense, peripheral
retinal elevations develop, most commonly in the inferotemporal quadrant, although less frequently other quadrants may be
involved. Retinal breaks may develop in either or both the inner or outer retinal layers, or the adjacent full thickness retina.
Retinal detachments associated with retinoschisis occur infrequently, but may develop when either an outer layer retinal
break alone or both inner and outer layer retinal breaks are present. Retinal detachments associated with senile
retinoschisis are rare and typically progress slowly; therefore, outer layer breaks may not routinely require prophylactic
treatment. Retinal cryotherapy, or scatter photocoagulation, may be used over the area of retinoschisis and outer layer
retinal hole formation if the detachment progresses. Typically, senile retinoschisis does not progress posteriorly. Attempts to
demarcate the area of retinoschisis by laser photocoagulation will not prevent posterior extension. Unlike in juvenile
retinoschisis, macular function remains normal.
Question 44 of 130
In a patient with postoperative, appositional choroidal effusion or hemorrhage following cataract surgery, which of the
following diagnostic tests is most useful?

Optical coherence tomography

Transillumination of the sclera

Fluorescein angiography

B-scan ultrasonography
Please select an answer
Feedback: In a patient with postoperative, appositional choroidal effusion or hemorrhage following cataract surgery, B-scan
ultrasonography is often helpful, and sometimes essential in making the diagnosis. Fluorescein angiography, optical
coherence tomography, and transillumination of the sclera are rarely helpful.

Question 45 of 130
The parents of a 2-year-old girl report that she has had "bobbing eyes" and light sensitivity since birth. In your office, the girl
shows good visual attention, but has bilateral pendular nystagmus and squints in bright light. The retina appears normal, but
the foveal reflex is blunted. Dark-adapted scotopic electroretinogram (ERG) responses are normal, but light-adapted
photopic signals are greatly diminished. No relatives are similarly affected. What condition does this patient most likely
have?

Achromatopsia

Congenital stationary night blindness (CSNB)

Leber's congenital amaurosis

Stargardt's disease
Please select an answer
Feedback: The early onset of photophobia and nystagmus point to a congenital cone dysfunction. The loss of photopic ERG
response, including photopic flicker response, and the patient's essentially normal rod function confirm the generalized cone
abnormality. While CSNB can reduce acuity and cause nystagmus, the congenital loss of night vision from rod system
abnormalities would cause an abnormal scotopic ERG. CSNB is frequently an X-linked recessive trait that affects males. In
Leber's congenital amaurosis, overall vision is very limited, because of generalized retinal dysfunction that causes loss of
both rod and cone ERG responses. Stargardt's disease becomes evident during the grade school and teenage years and
typically shows a nearly normal rod and cone ERG. Achromatopsia causes total color blindness from a congenital absence
of cone photoreceptors. It is autosomal recessive and rarely affects more than the current generation. Ultimate acuity ranges
from 20/100 to 20/200 and is best in dimmer light or with sunglasses.
Question 46 of 130
Which of the following statements does not accurately describe the use of indirect ophthalmoscopy to screen for retinopathy
of prematurity?

Screening should be repeated biweekly on neonates who demonstrate retinopathy of prematurity on the initial
examination.

Screening should be performed on all premature neonates with a birth weight of less than 1300g.

Screening should be performed prior to hospital discharge, or by 4 to 6 weeks of age.

Screening should be performed on all premature neonates of less than 30 weeks gestation.
Please select an answer
Feedback: Multiple risk factors have been associated with the development of retinopathy of prematurity in premature
neonates, including low birth weight, low gestational age, oxygen therapy apnea, sepsis, and others. Premature infants with
a birth weight of 1300g or less, of gestational age of 30 weeks or less, or who require supplemental oxygen, are particularly
at greater risk of developing retinopathy of prematurity. Initial examination of the peripheral retina by indirect
ophthalmoscopy in these neonates is recommended prior to hospital discharge, or by 4 to 6 weeks of age. Repeat retinal
examinations are performed every 2 weeks until the retina becomes fully vascularized, or retinopathy of prematurity is noted.
If retinopathy of prematurity develops, weekly examinations should be performed to watch for possible progression to
threshold disease.

Question 47 of 130
The early manifestations of idiopathic juxtafoveolar retinal telangiectasis does not include which of the following?

Pigmentary migration into the retina

Temporal graying of the macula

A diffuse pattern of late staining surrounding the fovea on fluorescein angiography

Asymmetric presentation
Please select an answer
Feedback: Many patients with idiopathic juxtafoveolar retinal telangiectasis present with symptoms in one eye only. The
early angiographic findings reveal late staining of the retina, often in an oval configuration. The earliest clinical feature is
graying of the retina temporal to the fovea. Only in the later stages of the disease does pigmentary migration occur. This
acquired form of idiopathic juxtafoveolar retinal telangiectasis usually presents in the middle-aged to elderly population. Most
patients retain good vision in at least one eye. The most common cause of visual loss is atrophy of the retinal pigment
epithelium. Choroidal neovascular membranes may also occur.
Question 48 of 130
An increased rate of proliferative vitreoretinopathy (PVR) has not been associated with which of the following?

Scleral buckling surgery, rather than closed vitrectomy, for the repair of retinal detachment

Cryoretinopexy

Vitreous hemorrhage

Large and/or multiple retinal breaks


Please select an answer
Feedback: Vitreous hemorrhage, cryoretinopexy, large or multiple retinal breaks, and vitrectomy surgery have all been
associated with an increased risk of developing PVR. Cryoretinopexy has been associated with both the dispersion of retinal
pigment epithelial cells into the vitreous, and the breakdown of the blood-retinal barrier (releasing serum components), both
of which may participate in the pathogenesis of PVR. Scleral buckling surgery, unlike vitreous surgery, has not been linked
to an increased risk of PVR, perhaps because it produces less breakdown of the blood-retinal barrier.

Question 49 of 130
A 64-year-old man is referred to you because of an elevated, brown choroidal mass involving the inferior quadrant of his left
eye. On echographic examination, the lesion shows a pattern highly consistent with malignant melanoma. A subsequent
metastatic workup is completely negative. Which of the following factors is not predictive of subsequent metastatic disease?

Location of the anterior tumor margin

Extrascleral Extension

Extension through Bruch's membrane

Cell type
Please select an answer
Feedback: A variety of factors have been shown to increase the risk of subsequent tumor-related death following
enucleation or radiation therapy for malignant melanoma. Tumors composed primarily of epithelioid cells (versus spindle A,
spindle B, or mixed-cell type) have a poor prognosis. Extrascleral extension of the tumor is also a grave prognostic sign.
Kaplan-Meier survival curves show a steadily worsening prognosis with every 2mm increase in the largest tumor dimension.
Tumors that involve the anterior choroid, particularly the ciliary body, have also been shown to have a poor prognosis.
Extension through Bruch's membrane is not predictive of later metastatic disease.

Question 50 of 130
Which of the following retinal defects is least likely to be associated with traumatic injury?

Round hole

Flap or horseshoe tear

Retinal dialysis

Choroidal rupture
Please select an answer
Feedback: Retinal dialysis, choroidal rupture, and horshoe retinal tears are commonly associated with severe ocular
trauma. Round retinal holes are commonly associated with atrophic processes or posterior vitreous detachment.
Question 51 of 130
A 35-year-old man complains of gradually decreasing vision in his right eye over at least the last year. His visual acuity is
20/200 OD and 20/20 OS. The anterior segment is normal in both eyes. Fundus examination of the right eye shows a heavy
accumulation of lipid in the posterior pole. The left fundus is completely normal. Which of the following would not be a
probable cause of the exudate?

Idiopathic central serous choroidopathy

Branch retinal vein occlusion

External beam irradiation

Von Hippel-Lindau syndrome


Please select an answer
Feedback: All of the listed conditions can produce unilateral lipid exudation in the posterior pole except idiopathic central
serous choroidopathy. Central serous choroidopathy, also referred to as retinopathy, is usually a self-limited disease. Visual
acuity is usually better than 20/200 and often can be shown to be better with a slight hyperopic correction. Laser
photocoagulation hastens recovery, but does not improve the final visual outcome compared with spontaneous recovery.

Question 52 of 130
Which of the following is the strongest indication for prophylactic treatment (cryopexy or laser surgery) to prevent
rhegmatogenous retinal detachment?

High myopia and lattice degeneration with new onset of floaters in a phakic patient

An asymptomatic flap tear in an eye with a cataract that is about to be rendered pseudophakic

An atrophic hole in a phakic patient whose other eye developed a retinal detachment

Lattice degeneration in a pseudophakic patient with a family history of retinal detachment


Please select an answer
Feedback: The subject of prophylaxis for rhegmatogenous retinal detachment is an area of some controversy among
vitreoretinal specialists. As with any surgical decision, the decision to treat must rest on an educated assessment of the risks
of treatment versus the risk of leaving the patient untreated, as well as on knowledge of how much the treatment can
actually reduce the risk of detachment. A thorough exploration is beyond the scope of this discussion, but factors that
prompt treatment are evidence of acute onset of symptoms and the presence of vitreous traction. Less compelling
indications are a history of detachment in the other eye and imminent cataract surgery, but the combination of the latter with
evidence of traction becomes a strong indication to treat.
Question 53 of 130
A 45-year-old man with a 4-month history of lung cancer complains of intense pain in the left eye. His oncologist states that
the patient has had a good response to radiation therapy. CT and MRI scans of the head and orbit are normal. Which of the
following statements best describes ocular tumors?

Echography is the most sensitive imaging technique for ocular tumors.

Metastatic tumors are the most common ocular tumors in adults.

Lung and breast cancer are the most common primary sources of metastatic ocular tumors.

Metastatic tumors are clinically distinguishable from melanomas.


Please select an answer
Feedback: Malignant melanoma of the choroid is the most common primary ocular tumor in adults. However, metastatic
tumors to the eye occur more commonly. In males they arise from the lung and gastrointestinal tract, whereas in females
they arise from the breast and lung. B-scan and standardized A-scan echography currently provide the most sensitive
means of imaging ocular and orbital tumors. Metastatic lesions tend to be highly reflective and may be located throughout
the uveal tract, within the extraocular muscles, or elsewhere in the orbit. Choroidal melanomas are characteristically low-
reflective using standardized A-scan echography. Ophthalmoscopically, metastatic lesions tend to be amelanotic. They may
be associated with an exudative retinal detachment and "shifting" subretinal fluid, multiple foci, "leopard-spot pigmentation,"
and a characteristic fluorescein pattern. Some metastatic lesions, particularly adenocarcinoma of the lung and carcinoid
tumors, may be indistinguishable echographically and ophthalmoscopically from malignant melanoma of the choroid.

Question 54 of 130
A 30-year-old woman complains of blurred central vision in the right eye for the past 3 days. Multiple chorioretinal lesions
are noted in both eyes. Which of the following findings is most consistent with a diagnosis of acute posterior multifocal
placoid pigment epitheliopathy (APMPPE)?

Symptoms in the other eye within 2 days

Subretinal hemorrhage in the macula and peripheral scotomas

Subretinal fluid and hemorrhage in the macula, plus vitreous cells

Vitreous cells and satellite lesions


Please select an answer
Feedback: Possible diagnoses, besides APMPPE, are presumed ocular histoplasmosis syndrome (POHS), recurrent
multifocal choroiditis (pseudo POHS), and toxoplasmosis. While all of these conditions have quite different clinical findings
and presentations, variants of them can sometimes be confusing. The typical POHS case has peripapillary atrophy of the
retinal pigment epithelium and peripheral "histo spots"; and, in the macula, choroidal neovascularization with subretinal fluid
and hemorrhage can occur. Bilaterality is common, but usually years elapse between macular involvement of the first eye
and that of the other eye. Vitreous cells are not seen in typical POHS. Recurrent multifocal choroiditis mimics POHS. It is
distinguished clinically from POHS by mild panuveitis and frequent recurrences, with the development of new fundus
lesions. Unilateral cases may become bilateral. Toxoplasmosis produces focal areas of chorioretinal inflammation, often
adjacent to previous areas of involvement (satellite lesions). Heavy vitreous cellular infiltration is the rule, and it is unusual to
have simultaneous bilateral involvement. APMPPE usually occurs following a viral illness and is usually bilateral, with rapid
involvement of both eyes once symptoms begin. Spontaneous recovery of vision is usual.
Question 55 of 130
Crystalline retinopathy will not arise from which of the following substances?

Heroin

Methoxyflurane

Canthaxanthine

Tamoxifen
Please select an answer
Feedback: Crystalline deposits in the retina are a toxic manifestation of tamoxifen, an anti-estrogen drug used in the
treatment of metastatic breast carcinoma; methoxyflurane, a nonflammable inhalant general anesthetic agent that may
produce oxalosis; and canthaxanthine, an oral skin-tanning agent marketed outside the United States. Talc retinopathy,
another form of crystalline retinopathy, occurs in drug abusers who inject multiple crushed tablets of Ritalin or methadone
intravenously; it does not occur from heroin use. The differential diagnosis of crystalline retinopathy also includes Bietti's
crystalline dystrophy, Sjogren-Larsson syndrome, nephropathic cystinosis, gyrate atrophy, and oxalosis.

Question 56 of 130
A 58-year-old woman has no particular complaints on first-time routine examination. She has not had a dilated fundus
examination before. Visual acuity is normal in each eye, but you find a peripheral area of retina that is thin and elevated in
the right eye. You would like to know whether the appearance of the retina is due to retinoschisis, or a rhegmatogenous
retinal detachment. Which of the following is least likely to indicate retinoschisis?

Atrophy of the underlying pigment epithelium

Hyperopia

Ophthalmoscopically a highly elevated, dome-shaped configuration

A scotoma on Goldmann visual field testing


Please select an answer
Feedback: In retinoschisis, the pigment epithelium under the involved retina is normal and there is no atrophy, demarcation
line, or subretinal strand formation, unless it is associated with a combined rhegmatogenous retinal detachment. Both
rhegmatogenous retinal detachment and retinoschisis produce scotomas, but the scotoma in retinoschisis is absolute, while
in rhegmatogenous retinal detachment it is relative. Retinoschisis is typically bilateral, dome-shaped, and seen more often in
hyperopic eyes.
Question 57 of 130
Choroidal hemangiomas are not often associated with which of the following findings?

Subretinal serous or lipid exudation

High internal reflectivity on A-scan echography

A subretinal neovascular membrane

Leakage of dye in the late phases of fluorescein angiography


Please select an answer
Feedback: Choroidal hemangiomas are benign vascular tumors that may arise as isolated lesions without other vascular
malformations, or as diffuse thickening of the choroid in association with Sturge-Weber syndrome, a nonfamilial condition
characterized by ipsilateral angiomatous lesions involving the face, brain, and uveal tract. In contrast to melanomas,
choroidal hemangiomas exhibit high internal reflectivity on A-scan echography. During fluorescein angiography, they fill
during the prearteriole and arteriole phases, and leak during the late phases. Serous or lipid exudation accounts for loss of
vision in many patients. Subretinal neovascular membranes rarely, if ever, develop from choroidal hemangiomas.

Question 58 of 130
A 67-year-old man complains that within the last 2 weeks he has had a severe loss of vision in the right eye. His visual
acuity is 20/400 OD and 20/25 OS. Examination of the anterior segments shows only mild nuclear sclerosis bilaterally.
Fundus examination of the right eye shows disc edema, dilated and tortuous retinal veins, and multiple superficial intraretinal
hemorrhages. Which of the following additional findings is least likely to be present in this patient?

Decreased a-wave amplitude on the electroretinogram

An afferent pupillary defect in the right eye

Intraocular pressure slightly above normal in both eyes

Cystoid macular edema seen on fluorescein angiography


Please select an answer
Feedback: This clinical picture describes a central retinal vein occlusion. Factors predisposing to this condition include
elevated intraocular pressure and systemic hypertension. Some leakage may be seen throughout the fundus, and cystoid
macular edema is common. If inner retinal ischemia is significant, it may be reflected in an afferent pupillary defect, or a
decreased b-wave on electrophysiologic testing. The a-wave is produced at the photoreceptors and is not affected by inner
retinal ischemia.
Question 59 of 130
About 24 hours after extracapsular cataract extraction and insertion of a posterior chamber intraocular lens, a 68-year-old
man is found to have reduced vision and a large afferent pupillary defect in the operated eye. The ocular media are clear.
The fundus examination shows whitening and mild intraretinal hemorrhage of the retina. Fluorescein angiography reveals
marked capillary nonperfusion of the major retinal arterioles. What are these findings consistent with?

Anterior ischemic neuropathy

Branch retinal artery occlusion

Phototoxicity from the operating microscope

Gentamicin toxicity
Please select an answer
Feedback: The whitening of the retina occurring from phototoxicity is transient and usually located inferior to the fovea,
because of the rotation of the globe from the superior rectus muscle bridle suture. Within hours, the retinal swelling resolves;
however, it often leads to a stippled retinal pigmentary disturbance. Acute retinal necrosis occurs from mistakenly injecting
gentamicin instead of miochol or other drugs intracamerally, or from accidental injection of gentamicin through the sclera at
the conclusion of surgery. Doses only slightly greater than 200 micrograms (subconjunctival injections are routinely 20
milligrams or 20,000 micrograms) may produce severe retinal vascular occlusion within minutes, and therefore attempts to
irrigate gentamicin from the vitreous cavity using pars plana vitrectomy techniques may be futile. Acute anterior ischemic
neuropathy is associated with disc swelling without retinal whitening. Branch retinal artery occlusion is not associated with
retinal hemorrhage, and the whitening of the retina is localized to the distribution of the affected branch retinal artery.

Question 60 of 130
Which of the following statements is most consistent with a 66-year-old man with asteroid hyalosis in his right eye
complaining of loss of vision?

Asteroid hyalosis and optic disc drusen are easily recognizable echographically.

The macula can readily be evaluated in an eye with asteroid hyalosis using the excitor and barrier filters of a fundus
camera.

Dense asteroid hyalosis seldom produces loss of vision or the sensation of vitreous floaters.

Asteroid bodies usually occur unilaterally and appear white in an aphakic or pseudophakic eye.
Please select an answer
Feedback: Asteroid hyalosis is unilateral in nearly 75% of cases and seems to be more prevalent in patients with diabetes.
It seldom results in any visual disturbance, although it may hinder evaluation of the ocular fundus. Asteroid bodies and optic
disc drusen exhibit characteristic patterns echographically. Because asteroid bodies do not emit fluorescent light, they are
invisible during fluorescein angiography, and therefore details of the posterior fundus can be readily photographed.
Question 61 of 130
A 70-year-old man comes to your office for the first time complaining of poor vision in his recently operated left eye. Your
initial examination reveals an early nuclear sclerotic cataract in the right eye, and pseudophakia in the left eye. His best-
corrected visual acuity is 20/70 OD, which you judge to be consistent with his cataract, and 20/60 OS. Which of the following
findings points most strongly toward retinal disease as a cause of the decreased acuity in the left eye?

A strongly positive afferent pupillary defect

Metamorphopsia on Amsler grid testing

Improvement of 3 lines of acuity with pinhole testing

Inability to recognize 8/10 pseudoisochromatic plates


Please select an answer
Feedback: While macular disease can produce a subtle afferent pupillary defect, a strongly positive afferent defect points
toward optic nerve disease. Similarly, profound loss of color vision in a patient with 20/60 vision is more commonly seen in
optic nerve dysfunction than in retinal disease. Some improvement may be noted on pinhole testing in cases of cystoid
macular edema, but usually only 1 line or less. Metamorphopsia is produced only by retinal disease.

Question 62 of 130
A fluorescein angiogram shows perifoveal capillary leakage in the early and mid-angiogram, and a petalloid or stellate
pattern in the latest frames. Which of the following statements does not accurately describe this retinal disease process?

Disc staining commonly occurs in the late frames of the fluorescein angiogram.

Hyperbaric oxygen has improved visual acuity in some patients.

Extracellular cystoid spaces within the outer plexiform layer are typically seen histopathologically.

Ultraviolet-blocking intraocular lenses have been shown statistically to improve visual outcome by reducing the
perifoveal capillary leakage.
Please select an answer
Feedback: Ultraviolet-blocking intraocular lenses have, in some studies, been shown to decrease the incidence of
fluorescein angiographically demonstrated cystoid macular edema. However, no statistically significant evidence has yet
been reported to support the hypothesis that the visual outcome is better with ultraviolet-blocking intraocular lenses.
Question 63 of 130
Which of the following is not a characteristic of diode-red laser light?

Less light scatter from vitreous hemorrhage than argon wavelengths

Strong absorption by hemoglobin

Ability to penetrate through lens nuclear sclerosis

Minimal absorption by macular xanthophyll


Please select an answer
Feedback: Diode red is not absorbed well by hemoglobin. Similarly, it is not absorbed, but penetrates well through lens
nuclear sclerosis and macular xanthophyll. Optical scattering is less from longer wavelengths than from shorter
wavelengths, making diode red relatively advantageous in terms of light scattering.

Question 64 of 130
You see a 26-year-old man with a 15-year history of insulin-dependent diabetes mellitus for the first time. His visual acuity is
20/20 OU. Fundus examination of the right eye reveals the vitreous to be clear. No neovascularization is visible on the disc
or elsewhere; however, there are numerous microaneurysms, venous beading, and rare cotton-wool spots. Examination of
the left fundus also reveals a clear vitreous with no blood present, but there is a frond of elevated neovascularization arising
from the disc, and another small patch of neovascularization away from the disc along the inferonasal arcade. If the diabetic
retinopathy were asymmetric between the eyes on initial presentation, what would the most likely explanation be?

A history of optic neuritis on the less involved side

Atherosclerotic carotid disease on the more involved side

A history of anisometropic hyperopia

Atherosclerotic carotid disease on the less involved side


Please select an answer
Feedback: Atherosclerotic carotid disease would be highly unusual in a 26-year-old patient, but if present it could be
involved on either side. Optic neuritis, along with any cause of optic atrophy, is relatively protective against diabetic
retinopathy.
Question 65 of 130
You see a 26-year-old man with a 15-year history of insulin-dependent diabetes mellitus for the first time. His visual acuity is
20/20 OU. Fundus examination of the right eye reveals the vitreous to be clear. No neovascularization is visible on the disc
or elsewhere; however, there are numerous microaneurysms, venous beading, and rare cotton-wool spots. Examination of
the left fundus also reveals a clear vitreous with no blood present, but there is a frond of elevated neovascularization arising
from the disc and another small patch of neovascularization away from the disc along the inferonasal arcade. Which of the
following statements best describes the right eye?

A fluorescein angiogram should be obtained to detect or rule out the presence of clinically significant macular edema.

If the patient's diabetic control has been poor, normalizing the blood glucose level and decreasing the glycosylated
hemoglobin will decrease the risk of progression to proliferative retinopathy.

If there is a small area of thickened retina, with its center 500 microns temporal to the center of the fovea, immediate
focal laser photocoagulation surgery can be considered.

A ring of exudate, 500 microns in diameter that extends to within 1disc diameter of the center of the macula, can be
considered an indication for immediate focal laser photocoagulation.
Please select an answer
Feedback: The Early Treatment Diabetic Retinopathy Study shows that immediate treatment, even with 20/20 acuity, is
preferable to deferred treatment. The diagnosis of clinically significant macular edema is a clinical decision based on careful
examination of the fundus with biomicroscopy, usually with a fundus contact lens. Fluorescein leakage alone does not meet
the definition of clinically significant macular edema. The diagnosis depends on documenting any of the following: (1)
thickening of the retina at or within 500 microns of the center of the macula; (2) hard exudates at or within 500 microns of the
center of the macula, if associated with thickening of the adjacent retina (not residual hard exudates remaining after the
disappearance of retinal thickening); (3) a zone or zones of retinal thickening 1disc area or larger, any part of which is within
1disc diameter of the center of the macula. The Early Treatment Diabetic Retinopathy Study concludes that treatment of
eyes with clinically significant macular edema is preferable to no treatment during the follow-up period, but does not
specifically address the question of when treatment should be initiated, ie, immediate treatment or deferral if visual acuity is
still 20/20. The role of glucose control in the progression of diabetic retinopathy remains unclear. In some cases, rapid
improvement of control has been associated with accelerated progression of retinopathy.

Question 66 of 130
Metamorphopsia on Amsler grid testing is least compatible with which of the following?

Geographic atrophy of the retinal pigment epithelium

Chronic inferior rhegmatogenous retinal detachment up to the fovea

Serous and hemorrhagic detachment of the neurosensory retina in the macula

Serous detachment of the pigment epithelium in the macula


Please select an answer
Feedback: Geographic atrophy without subretinal or subretinal pigment epithelial fluid may produce scotomas, but typically
not true metamorphopsia. All the other conditions listed commonly produce metamorphopsia.
Question 67 of 130
You see a 26-year-old man with a 15-year history of insulin-dependent diabetes mellitus for the first time. His visual acuity is
20/20 OU. Fundus examination of the right eye reveals the vitreous to be clear. No neovascularization is visible on the disc
or elsewhere; however, there are numerous microaneurysms, venous beading, and rare cotton-wool spots. Examination of
the left fundus also reveals a clear vitreous with no blood present, but there is a frond of elevated neovascularization arising
from the disc and another small patch of neovascularization away from the disc along the inferonasal arcade. Peripheral
retinal photocoagulation is performed on the left eye of the patient, with the placement of about 1400 spots. Over the next
month, neovascularization is observed to have regressed. The patient is then lost to follow-up for several months, but
eventually returns stating that on awakening 6 weeks ago he became aware of profoundly decreased vision in the right eye.
Examination of the right eye reveals only hand-motion vision, and examination of the fundus is not possible because of
vitreous blood. Which of the following is not an appropriate management of the right eye?

Prompt vitrectomy if ultrasound examination shows unequivocally that the peripheral retina is detached

Observation for at least 6 months to allow for spontaneous clearing of vitreous blood if all tests demonstrate no retinal
detachment

Prompt vitrectomy if ultrasound examination shows clearly that the macula is detached

Strict bed rest with bilateral eye patches for 24 hours


Please select an answer
Feedback: The Diabetic Retinopathy Vitrectomy Study demonstrated that the outcome of early vitrectomy is significantly
better than that of deferred surgery for severe vitreous hemorrhage, even when the retina is attached, in insulin-dependent
diabetic patients. If the macula is clearly detached, however, prompt surgery is appropriate, regardless of the type of
diabetes or the age of onset. If the status of the retina cannot be assessed visually, bed rest with immobilization of the eyes
to allow settling of the blood may be of help.

Question 68 of 130
Which of the following findings is the least urgent indication to obtain a fluorescein angiogram in a 67-year-old woman who
has slightly decreased vision in one eye?

An isolated subretinal hemorrhage adjacent to the fovea

Scattered exudates limited to the central macula

Multiple intraretinal hemorrhages involving an arcuate patch of retina, including the macula

Metamorphopsia on Amsler grid testing


Please select an answer
Feedback: The description of intraretinal hemorrhages in an arcuate pattern suggests a branch retinal vein occlusion.
Although fluorescein angiography may be indicated with this condition, there is no real urgency, because most branch vein
occlusions show spontaneous improvement with time. At the time of initial evaluation, observation only is usually
appropriate. Unlike branch vein occlusion, subretinal or choroidal neovascularization requires prompt diagnosis and, when
possible, treatment. The three cardinal signs of choroidal neovascularization are subretinal or intraretinal hemorrhage,
subretinal or subretinal pigment epithelial serous fluid, and intraretinal exudate. The finding of any one of these is an
indication for obtaining a fluorescein angiogram promptly.
Question 69 of 130
Which of the following most strongly correlates with a diagnosis of pseudophakic cystoid macular edema?

A 10-year history of diet-controlled diabetes mellitus and no visible diabetic retinopathy

A loud carotid bruit on the left side

A strand of vitreous to the cataract wound and a peaked pupil

A history of chronic glaucoma controlled with pilocarpine


Please select an answer
Feedback: Vitreous to the wound is a cause of postoperative cystoid macular edema, chronic inflammation, and
predisposition to retinal detachment. While venous occlusive disease related to glaucoma (central vein occlusion) and ocular
ischemia related to carotid disease can produce cystoid changes in the macula, they are not commonly associated with
typical pseudophakic macular edema. A subretinal hemorrhage is highly suggestive of choroidal neovascularization due to
age-related macular degeneration, although an occasional intraretinal hemorrhage, or microaneurysm, may be seen in
association with pseudophakic cystoid macular edema.

Question 70 of 130
You see a 26-year-old man with a 15-year history of insulin-dependent diabetes mellitus for the first time. His visual acuity is
20/20 OU. Fundus examination of the right eye reveals the vitreous to be clear. No neovascularization is visible on the disc
or elsewhere; however, there are numerous microaneurysms, venous beading, and rare cotton-wool spots. Examination of
the left fundus also reveals a clear vitreous with no blood present, but there is a frond of elevated neovascularization arising
from the disc and another small patch of neovascularization away from the disc along the inferonasal arcade. Which of the
following statements would be most accurate about the left eye of the patient?

Over the next 5 years, the risk of severe visual loss can be reduced by at least 50% with peripheral retinal
photocoagulation.

If there is also clinically significant macular edema, peripheral retinal photocoagulation can be expected to reduce it.

According to criteria established by the Diabetic Retinopathy Study, the patient does not have high-risk characteristics
yet, because there is no vitreous hemorrhage.

Xenon-arc photocoagulation is less effective than argon laser surgery in reducing the risk of severe visual loss.
Please select an answer
Feedback: The Diabetic Retinopathy Study shows that with the treatment protocol employed, the risk of severe visual loss
is reduced by at least 50% over the period of observation. However, improvement in macular edema is not associated with
peripheral retinal treatment. Under the protocols tested in major trials, the treatment of high-risk proliferative retinopathy is
not directed at specific targets, but rather at the periphery and midperiphery, sparing the macular area. High-risk
characteristics, as defined by the Diabetic Retinopathy Study, include (1) neovascularization of the disc (NVD) greater than
114 to 113 of the disc area; (2) vitreous or preretinal hemorrhage associated with less extensive NVD, or with
neovascularization elsewhere (NVE), 112 of the disc area or more in size. Xenon-arc photocoagulation was found to have a
similar effect to that of argon laser photocoagulation in reducing severe visual loss, but was associated with slightly
increased side effects.
Question 71 of 130
In which situation is optical coherence tomography most accurate?

Proper scanning module alignment

Undilated pupil

Media opacification

Unsteady patient fixation


Please select an answer
Feedback: Accurate optical coherence tomography requires proper scanning module alignment. Miotic pupils, media
opacification, and unsteady patient fixation will reduce the accuracy of optical coherence tomography.

Question 72 of 130
Which one of the following represents a relevant aspect of the epidemiology of juvenile retinoschisis?

Outer retinal lamina breaks are more common than inner lamina breaks.

Retinal detachment occurs in 90-95% of eyes.

Female carriers can be readily clinically identified.

50% of those with foveal radiating retinal folds also have peripheral retinoschisis.
Please select an answer
Feedback: In juvenile retinoschisis, 50% of those with foveal radiating retinal folds also have peripheral retinoschisis.
Retinal detachment occurs in 5-20%. Retinal breaks may develop in inner lamina (75%) or outer lamina (13%). Female
carriers cannot be clinically identified.

Question 73 of 130
Which one of the following is contraindicted in the evaluation of a possible intraocular foreign body?

Magnetic resonance imaging (MRI)

Orbital X-rays

Computed tomographic (CT) scanning

Ultrasonography
Please select an answer
Feedback: The magnetic field utilized in performing magnetic resonance imaging could result in movement of a ferrous
intraocular foreign body, resulting in further tissue damage. The other imaging modalites listed do not pose this threat.
Question 74 of 130
Which of the following describes a pertinent clinical feature of acute retinal necrosis?

Occlusive venous vasculopathy

Peripheral retinal necrosis with discrete borders

Optic neuropathy is uncommon

Rapid progression despite antiviral therapy


Please select an answer
Feedback: Acute retinal necrosis typically manifests peripherally with discrete borders. Progression of the necrosis may be
halted with antiviral therapy. It results in an occlusive arteriolar vasculopathy. It is commonly associated with optic
neuropathy.

Question 75 of 130
Which one of the following represents a risk factor for the development of atrophic retinal holes?

Family history of atrophic holes or lattice degeneration

Young age

Hyperopia

Macular degeneration
Please select an answer
Feedback: Atrophic retinal holes are associated with a family history of such holes, or of lattice degeneration. In addition,
they are associated with increasing age and axial myopia. There is no association with macular degeneration.

Question 76 of 130
Which one of the following best describes the maculopathy of chloroquine and its derivatives?

There is no association with hepatic or renal failure.

Most cases occur between 20 and 50 years of age and more commonly in females.

Most commonly occurs in Caucasians.

Obesity is a risk factor.


Please select an answer
Feedback: Most cases of maculopathy related to chloroquine and its derivatives occur between 20 and 50 years of age and
more commonly in females. Lean patients are at greater risk per dose, because these drugs are not retained by fatty tissue.
Liver or renal failure increases risk, because these drugs are cleared by these organ systems. There is no racial
predilection.
Question 77 of 130
Which one of the following represents the most common cause of a spontaneous vitreous hemorrhage?

Retinal tear without detachment

Undiagnosed proliferative diabetic retinopathy

Rhegmatogenous retinal detachment

Posterior vitreous detachment


Please select an answer
Feedback: The most common cause of a spontaneous vitreous hemorrhage is undiagnosed proliferative diabetic
retinopathy. Although the other conditions listed may also result in a spontaneous vitreous hemorrhage, they are less
frequent causes of this entity.

Question 78 of 130
What is a pertinent element in the history of a patient with suspected albinism?

Absence of nystagmus

Another family member affected

Onset in adulthood

Brown or dark colored eyes


Please select an answer
Feedback: As oculocutaneous albinism is an autosomal recessive disorder, other family members may be affected.
Associated findings include childhood onset, nystagmus, and hypopigmented irides.

Question 79 of 130
Which one of the following is a symptom of central serous chorioretinopathy?

Pain

Peripheral visual field loss

Photophobia

Metamorphopsia
Please select an answer
Feedback: Metamorphopsia is a common presenting symptom of central serous chorioretinopathy. Pain, peripheral visual
field loss, and photophobia are not typically associated with this condition.
Question 80 of 130
Which one of the following is a risk factor for hypertensive retinopathy?

African-American ethnicity

Young age

Compliance with antihypertensive therapy

Malnutrition
Please select an answer
Feedback: Risk factors for hypertensive retinopathy include African-American ethnicity, advanced age, obesity, and
noncompliance with antihypertensive therapy.

Question 81 of 130
A patient develops a hemorrhagic choroidal detachment following cataract surgery. Their intraocular pressure remains in the
normal range. Which of the following developments is most likely?

Proliferative vitreoretinopathy

Posterior uveitis

Spontaneous anatomical resolution

Rhegmatogenous retinal detachment


Please select an answer
Feedback: In a patient with a hemorrhagic choroidal detachment and normal intraocular pressure, spontaneous anatomical
resolution is the most likely outcome.

Question 82 of 130
What is the most frequent complication associated with pars plana vitrectomy and membrane peeling?

Accelerated cataractogenesis

Glaucoma

Retinal pigment epithelial disturbance

Retinal tear
Please select an answer
Feedback: Cataract is the most frequent complication associated with pars plana vitrectomy for phakic patients. Accelerated
cataractogenesis leads to cataract surgery in 50% of patients within 2 years of vitrectomy. Glaucoma, retinal tears, and
retinal pigmented epithelial disturbances occur at a frequency of less than 5%.
Question 83 of 130
A 28-year-old man with acquired immunodeficiency syndrome (AIDS) presents with isolated retinal nerve fiber layer infarcts.
Which of the following statements regarding the patient's condition is most accurate?

The lesions represent pneumocystis choroiditis.

The lesions represent CMV retinitis.

The lesions represent HIV retinopathy.

The lesions represent acute retinal necrosis.


Please select an answer
Feedback:

HIV retinopathy is the most common cause of isolated retinal nerve fiber layer infarcts in patient with AIDS. Small areas of
retinitis from CNV may simulate cotton-wool spots, but this is not a typical presentation. Pneumocystis choroiditis lesions
can be differentiated from more superficial retinal lesions on examination. Acute retinal necrosis cannot, by definition, be
present in AIDS.

Question 84 of 130
What are the characteristics of a Stage 3 macular hole?

Small, perifovesl hole with no posterior vitreous separation

Full thickness hole with surrounding subretinal fluid and no posterior vitreous separation

Re-opened Stage 2 macular hole

Full thickness hole with surrounding subretinal fluid and a posterior vitreous separation
Please select an answer
Feedback: The staging system applies only to idiopathic macular holes (IMH), not traumatic or disorder associated macular
holes. A Stage 3 idiopathic macular hole is characterized by adjacent annulus of subretinal fluid without a posterior vitreous
separation. A Stage 4 macular hole is a full thickness hole with a posterior vitreous separation. A Stage 2 macular hole is
defined as a perifoveal or "can-opener"-like hole. Occasionally, surgical repair of idiopathic macular holes will reopen
spontaneously or following surgical intervention. However, hole re-opening is not included in the staging system.

Question 85 of 130
Which of the following is the primary cause of visual loss following branch retinal vein occlusion?

Retinal neovascularization without vitreous hemorrhage

Collateral vessel formation

Macular edema

Refractive change
Please select an answer
Feedback: The primary cause of visual reduction following branch retinal vein occlusion is macular edema. Retinal
neovascularization is uncommon, but may be associated with vitreous hemorrhage. Collateral vessel formation may be
associated with visual improvement from normalization of venous luminal pressure. Refractive changes, if found, are
typically secondary to macular edema.
Question 86 of 130
Which of the following features of age-related macular degeneration (AMD) was associated with an increased risk of
choroidal neovascularization development in the AREDS study?

Central retinal pigment epithelial atrophy

A single, large druse

Bilateral subfoveal geographic atrophy

Numerous small drusen


Please select an answer
Feedback: In the AREDS study, the presence of at least one large druse (>250 microns) correlated with an increased risk of
choroidal neovascularization (CNV) development. Numerous small (<125 microns), geographic atrophy, and diffuse retinal
pigmented atrophy did not have an association with CNV.

Question 87 of 130
What is the most common cause of visual loss in patients with nonproliferative diabetic retinopathy?

Diabetic macular edema

Macular pigmentary disturbance

Traction retinal detachment

Vitreous hemorrhage
Please select an answer
Feedback: Diabetic macular edema is the most common cause of visual loss in patients with nonproliferative diabetic
retinopathy. Vitreous hemorrhage and traction retinal detachment more typically are associated with proliferative diabetic
retinopathy. Macular pigmentary disturbance may be a manifestation of macular edema, but not the primary cause of visual
loss.

Question 88 of 130
Which of the following is an appropriate management option for a perforating scleral injury?

CT scan

MRI scan

Avoidance of dilation

Scleral depression
Please select an answer
Feedback: An appropriate management option for a scleral perforating injury is a CT scan to rule out the presence of a
retained intraocular foreign body (IOFB). MRI is contraindicated in the setting of possible IOFB. Scleral depression is
contraindicated as intraocular contents could be extruded. All traumatized eyes should undergo dilated examination in the
clinic or operating room to evaluate for associated injuries such as traction retinal detachment or retinal incarceration.
Question 89 of 130
Which of the diagnostic studies is indicated in the evaluation of age-related macular degeneration to detect the presence
of choroidal neovascularization?

Fluorescein angiography

Corneal topography

Computerized axial tomography

Magnetic resonance imaging


Please select an answer
Feedback:

Fluorescein angiography is indicated in the evaluation of age-related macular degeneration to detect the presence
of choroidal neovascularization.

Question 90 of 130
Which of the following conditions may exacerbate diabetic macular edema?

Hyperthyroidism

Systemic hypertension

Hypokalemia

Atrial fibrillation
Please select an answer
Feedback:

Systemic hypertension may exacerbate diabetic macular edema. Hyperthyroidism, which is unassociated with hypertension,
is not known to worsen diabetic complicatitons. Similarly, hypokalemia and atrial fibrillation are not appreciated to aggrevate
diabetic macular edema.

Question 91 of 130
Diabetic retinopathy is the leading cause of pemanent blindness in which of the following age catagories?

Children (12 years and younger)

Retired adults (age 65 years and older)

Adolescents (age 13-19 years)

Working adults (age 20-64 years)


Please select an answer
Feedback: Diabetic retinopathy is the leading cause of permanent blindness in working age adults, ages 20-64 years. Age-
related macular degeneration is the leading cause of permanent blindness in adults aged 65 and older. Vision-threatening
diabetic retinopathy is rare before the age of 20 years.
Question 92 of 130
Which of the following interventions is the most appropriate for treating an acute central retinal artery occlusion?

Anterior chamber paracentesis

Aspirin

Ocular antihypertensive therapy

Panretinal laser photocoagulation


Please select an answer
Feedback: Although not universally effective, anterior chamber paracentesis should be considered in the management of
the acute phase of central retinal artery occlusion. Ocular massage has also be proposed to force retrograde or antegrade
displacement of emboli. Panretinal photocoagulation and ocular antihypertensives are not useful for CRAO. Aspirin has
been advocated as antiplatelet therapy to lower risk of thrombotic, but not to treat acute embolic occlusions.

Question 93 of 130
What organisms are responsible for acute retinal necrosis (ARN) and progressive outer retinal necrosis (PORN)?

Rubella and rubeola

Varicella-zoster and herpes simplex

Borrelia bergdorferi and Bartonella henselae

Mycobacterium tuberculosis and Treponema pallidum


Please select an answer
Feedback: The organisms most commonly responsible for acute retinal necrosis (ARN) and progressive outer retinal
necrosis (PORN) are varicella-zoster and herpes simplex. Rubella and rubeola do not cause acute retinitis in post-natal
humans. Borrelia bergdorferi is the causitive organism of Lyme disease. Bartonella henselae is the causitive organism of
cat-scratch disease. Mycobacterium tuberculosis and Treponema pallidum may cause retinitis and/or choroiditis, but are not
the causitive organisms of acute retinal necrosis (ARN) and progressive outer retinal necrosis (PORN).

Question 94 of 130
In a phakic patient, for which of the following retinal breaks is treatment most urgent?

Round hole in lattice degeneration

Symptomatic retinal tears

Asymptomatic retinal tears

Asymptomatic operculated retinal tears


Please select an answer
Feedback: Treatment is most indicated in a phakic eye for symptomatic retinal tears due to the risk of retinal detachment
associated with this type of retinal break. As the incidence of clinical retinal detachment is low in association with round
holes in lattice degeneration as well as with asymptomatic retinal tears with or without an operculum, treatment is generally
not indicated for these types of breaks.
Question 95 of 130
What mechanism, prognosis or treatment applies to asymptomatic atrophic retinal holes?

Usually caused by blunt trauma

Requires demarcation treatment

Low likelihood to progress to symptomatic retinal detachment

Highly likely to progress to asymptomatic retinal detachment.


Please select an answer
Feedback:

As asymptomatic atrophic retinal holes have a low incidence of progression to retinal detachment; treatment is not usually
indicated. Atrophic retinal holes are rarely caused by traumatic. Blunt trauma is more closely associataed with retinal
dialysis.

Question 96 of 130
Which of the following conditions is most commonly associated with posterior vitreous detachment (PVD)?

Axial myopia

Emmetropia

Astigmatism

Hyperopia
Please select an answer
Feedback: Of the refractive errors listed, axial myopia is most commonly associated with posterior vitreous detachment
(PVD). This is thought to be related to the increased axial length present in this condition. The increased incidence of PVD
correlates with the increased incidence of retinal tears and detachment in this population.

Question 97 of 130
Vision threatening ocular toxoplasmosis is commonly treated with which oral antibiotic regimen?

Folinic acid

Pyrimethamine

Vancomycin

Penicillin
Please select an answer
Feedback:

Pyrimethamine has historically been the most commonly used oral antibiotic in treating vision-threatening ocular
toxoplasmosis. Other options include sulfadiazine or triple-sulfa, azithromycin, and clindamycin. Folinic acid is not an
antibiotic.
Question 98 of 130
For non-exudative age-related macular degeneration, which of the following conditions is associated wtih a risk of vision
loss?

One large druse

Diabetes mellitus

Congestive heart failure

Controlled systemic hypertension


Please select an answer
Feedback: In the AREDS Study, the presence of 1 or more large drusen (greater than 250 microns) increased the risk of
visual loss. Controlled systemic hypertension, congestive heart failure, and diabetes mellitus do not represent direct
negative risk factors for visual loss in non-exudative age-related macular degeneration.

Question 99 of 130
For a patient with a diabetic mid-peripheral traction retinal detachment, what complication is of primary concern when
applying initial panretinal photocoagulation?

Retinal neovascularization

Rhegmatogenous retinal detachment

Traction macular detachment

Cortical cataract
Please select an answer
Feedback: In proliferative diabetic retinopathy, panretinal photocoagulation may result in the contraction of fibrovascular
tissue, exacerbating tractional retinal detachment, resulting in macular detachment. Although this may also result in retinal
breaks and combined traction-rhegmatogenous retinal detachment, this is uncommon. Panretinal photocoagution reduces
retinal neovascularization and has no known direct impact on cortical cataract formation.

Question 100 of 130


Which of the following objects may cause a perforating ocular injury?

Golf ball

Paint ball

Metal-on-metal projectile

Fist
Please select an answer
Feedback: An ocular perforating injury has both entry and exit wounds. This requires two penetrations of the sclera. Blunt
trauma such as a fist, golf ball or paint ball will cause a scleral rupture, but are not capable of creating a perforating ocular
injury.
Question 101 of 130
Although a number of adjunctive procedures remain controversial in the surgical management of macular holes, which of the
following steps is crucial to high hole-closure rates?

Supine positioning

Silicone oil tamponade

Removing posterior vitreous hyaloid

Removing the retinal internal limiting membrane


Please select an answer
Feedback: The surgical management of macular holes involves pars plana vitrectomy, posterior hyaloid removal with or
without internal limiting membrane dissection, and fluid-gas exchange. Internal limiting membrane dissection, although
commonly performed, is not always necessary to achieve hole closure. Silicone oil tamponade is occasionally used in
patients when postoperative head positioning is impossible or postoperative air travel is necessary. There is no role for
supine positioning in macular hole surgery.

Question 102 of 130


Which of the following features is characteristic of moderate nonproliferative diabetic retinopathy?

Venous beading in all quadrants

Retinal microaneurysms

Retinal neovascularization

Vitreous hemorrhage
Please select an answer
Feedback: Retinal microaneurysms are a clinical finding in moderate nonproliferative diabetic retinopathy. Venous beading
in 2 or more quadrants is characteristic of severe nonproliferative diabetic retinopathy. Retinal neovascularization and
vitreous hemorrhage is seen in proliferative diabetic retinopathy

Question 103 of 130


Which one of the following is a risk factor for diabetic macular edema?

Systemic carbonic anhydrase inhibitors

Topical corticosteroid therapy

Elevated intraocular pressure

Elevated systemic blood pressure


Please select an answer
Feedback: Systemic hypertension is a risk factor for the development of diabetic macular edema. Elevated intraocular
pressure, topical corticosteroid therapy, and systemic carbonic anhydrase inhibitor use have not been shown to be risk
factors for the development of diabetic macular edema.
Question 104 of 130
An giant retinal tear is defined as a tear extending circumferentially for at least how many clock hours?

2
Please select an answer
Feedback: A giant retinal tear is defined as a tear extending 90 degrees or more circumferentially about the retinal
periphery. This equates to a circumferential extent of 3 or more clock hours.

Question 105 of 130


A 57-year-old woman notes recent-onset visual loss. An ocular coherence tomogram (OCT) of the macula is shown. What is
the preferred management?

Intravitreal triamcinolone acetonide injection

Intravitreal injection of autologous serum

Pars plana vitrectomy, posterior hyaloid removal, fluid-gas exchange

Pars plana vitrectomy, perifoveal laser photocoagulation


Please select an answer
Feedback: The OCT image demonstrates a full-thickness macular hole. The surgical management of macular holes
involves pars plana vitrectomy, posterior hyaloid removal with or without internal limiting membrane dissection, and fluid-gas
exchange. Intravitreal triamcinolone acetonide injection, intravitreal injection of autologous serum, and perifoveal laser
photocoagulation are not currently used in surgical management of macular holes.

Question 106 of 130


Which of the following is associated with the diagnosis of an idiopathic macular hole?

A 75% risk of developing similar condition in contralateral eye over 3 years.

More common in males.

Most prevalent in patients 50 - 80 years old.

Poor visual prognosis if not treated within 6 weeks of onset.


Please select an answer
Feedback:

Idiopathic macular holes are most prevalent in female patients 50 - 80 years old. The risk of fellow eye involvement has
been reported in the range of 1-25%. Macular holes successfully repaired within 6 months of onset have a high likelihood of
visual improvement.
Question 107 of 130
Which of the following tests is the most specific and sensitive for diagnosing an idiopathic macular hole?

Positive Watzke-Allen slit-beam test

B-scan ultrasonography

Optical coherence tomography (OCT)

Fluorescein angiography
Please select an answer
Feedback: The high resolution of current-generation optical coherence tomography makes this test more specific and
sensitive for diagnosing an idiopathic macular hole than the other techniques.

Question 108 of 130


A patient presents with diabetic macular edema with extensive lipid deposition in the macula. Which of the following
laboratory evaluations would be most appropriate?

C-reactive protein

Serum protein electrophoresis

Plasma homocysteine

Plasma lipid profile


Please select an answer
Feedback: Elevated serum lipid levels may be associated with an increased risk of diabetic macular edema with or without
intraretinal lipid deposition. Hyperhomocysteinemia, serum dysproteinemias, and elevated serum C-reactive protein levels
have not been shown to represent direct risk factors for diabetic macular edema.

Question 109 of 130


What condition is commonly found in association with vitreomacular traction syndrome?

Retinal pigment epithelial tear

Granulomatous inflammation

Choroidal neovascular membrane

Macular edema
Please select an answer
Feedback: The vitreomacular traction syndrome results from the incomplete separation of the posterior vitreous hyaloid
from the macula with resulting macular distortion and edema. The condition is idiopathic and is not typically associated with
retinal pigment epithelial tears, granulomatous inflammation, or choroidal neovascularization.
Question 110 of 130
Which of the following is the most common cause of visual loss in patients with nonproliferative diabetic retinopathy?

Vitreous hemorrhage

Diabetic macular edema

Neovascular glaucoma

Retinal neovascularization
Please select an answer
Feedback: Diabetic macular edema is the most common cause of visual loss in patients with nonproliferative diabetic
retinopathy. Retinal neovascularization, vitreous hemorrhage, and neovascular glaucoma are all findings in proliferative
diabetic retinopathy.

Question 111 of 130


Retinal arteriolar macroaneurisms are most frequently associated with what extra-ocular disorder?

Atrial fibrillation

Hyperthyroidism

Hypertension

Renal insufficiency
Please select an answer
Feedback: Retinal arteriolar macroaneurysm is highly associated with systemic hypertension. There is no known
association between retinal arteriolar macroaneurysm and hyperthyroidism, atrial fibrillation, or renal insufficiency.

Question 112 of 130


A 1-year-old boy presents with bilateral retinoblastoma. His father had an eye enucleated for retinoblastoma. What is the
probability of each of the child's siblings developing a retinoblastoma?

100%

5%

25%

45%
Please select an answer
Feedback: If one parent had unilateral retinoblastoma, the probability of the parent's first child having retinoblastoma is 7-
15%. If one parent had bilateral retinoblastoma, the probability of the parent's first child having retinoblastoma is 45%. If a
parent had a retinoblastoma and one sibling has a retinoblastoma (demonstrating autosomal dominance) the probability of
the each child's sibling will develop a retinoblastoma is 45%. Retinoblastoma has a 90% penetrance lowering the probabiity
from 50 to 45% for affected family members.
Question 113 of 130
A 7-year-old child presents with leukocoria. Based upon it incidence in this setting, what is the most likely posterior
segment lesion to be noted on ophthalmoscopic examination?

Idiopathic sclerochoroidal calcification

Ocular toxocariasis

Choroidal neovascularization

Asteroid hyalosis
Please select an answer
Feedback: Of the choices given, ocular toxocariasis is the most likely diagnosis. Retinoblastoma is unlikely to present with
leukocoria at this advanced age. Asteroid hyalosis is uncommon in youth and does not present with a retinal mass lesion.
Idiopathic sclerochoroidal calcification and choroidal neovascularization are uncommon in youth and do not typically present
with leukocoria.

Question 114 of 130


A 42-year-old man has a total retinal detachment and a circumferential, 150-degree peripheral retinal tear with an inverted
flap. What surgical approach would be most appropriate?

Scleral buckle with intravitreal injection of SF6 gas

Pars plana vitrectomy, retinal reattachment with perfluorocarbon liquid, laser photocoagulation, and complete fluid-gas
exchange

360-degree peripheral laser photocoagulation

Intravitreal injection 0.3 mL of SF6 gas


Please select an answer
Feedback: Retinal detachments from giant retinal tears, tears of greater than 90 degrees, have a high redetachment rate
due to proliferative vitreoretinopathy. Because of the high risk of failure the preferred approach would usually include pars
plana vitrectomy, use of perfluorocarbon liquid, laser photocoagulation demarcation and complete fluid-gas exchange. The
other options, performed alone, would not be likely to temporarily flatten or reattach the retina.

Question 115 of 130


Endophthalmitis following cararact surgery is most commonly caused by what group of bacteria?

Gram-negative rods

Gram-positive coagulase-negative micrococci

Gram-positive diplococci

Atypical mycobacteria
Please select an answer
Feedback: Gram-positive coagulase-negative micrococci are the most common organisms isolated in endophthalmitis
following cataract surgery. Gram-negative organisms, atypical mycobacteria, and gram-positive diplococci are less
commonly associated with endophthalmitis in this setting.
Question 116 of 130
What feature is a risk factor for the development of age-related macular degeneration?

Family history

Dark iris color

Obesity

Middle-Eastern ethnicity
Please select an answer
Feedback: Family history has been identified as a risk factor for the development of age-related macular degeneration;
obesity, Middle-Eastern ethnicity, and dark iris color have not.

Question 117 of 130


What systemic condition, which is associated with angioid streaks, may also present with blue sclera?

Von Hippel-Lindau syndrome

Wyburn-Mason syndrome

Sturge-Weber syndrome

Ehlers-Danlos syndrome
Please select an answer
Feedback: Angioid streaks and blue sclera are both found in those affected by Ehlers-Danlos syndrome. Angiod streaks
may also be associated with sickle cell disease, pseudoxanthoma elsticum and Paget's disease of bone. The is no known
association between angioid streaks or blue sclera and the Sturge-Weber, Von Hippel-Lindau, or Wyburn-Mason
syndromes.

Question 118 of 130


Panretinal photocoagulation is indicated in central retinal venous occlusion (CRVO) for what complication?

Intraretinal hemorrhage

Macular edema

Collateral vessels

Iris neovascularization
Please select an answer
Feedback: Panretinal photocoagulation is indicated to treat anterior segment neovascularization complicating central retinal
venous occlusion. PRP has not shown a benefit in treating macular edema, intraretinal hemorrhage, and would not be
expected to have any effect on collateral vessels
Question 119 of 130
What feature found in embolic central retinal artery occlusion (CRAO) is helpful in confirming the diagnosis?

Jaw claudication

Retinal pallor

Ocular pain

Subacute onset
Please select an answer
Feedback: Embolic central retinal artery occlusion (CRAO) causes retinal pallor which is often appreciated as a "cherry red
spot" due to relative retinal translucency at the fovea. However, retinal pallor is helpful to confirm a CRAO, but is not
specific for embolic CRAO. Pain, intermittant or subacute onset and jaw claudication suggest an inflammatory cause of
central artery occlusion. A minority of CRAOs are due to temporal arteritis, which may present with bilateral CRAO. A stat
CRP and/or sedimentation rate should be obtained if an emboli or other feature confirming embolic CRAO cannot be
confirmed on initial examination.

Question 120 of 130


What is the most common complication from branch retinal venous occlusion (BRVO)?

Macular edema

Neovascular glaucoma

Choroidal neovascularization

Subretinal hemorrhage
Please select an answer
Feedback: Macular edema is the most common complication of branch retinal venous occlusion (BRVO). Choroidal
neovascularization, subretinal hemorrhage, and neovascular glaucoma are not commonly associated with BRVO.

Question 121 of 130


What is the most likely diagnosis for the lesion shown?

Isolated choroidal hemangioma

Retinal angioma

Racemose aneurysm

Retinal cavernous hemangioma


Please select an answer
Feedback:

The image demonstrates a retinal capillary hemangioma or angioma typical of those occuring in von Hippel-Lindau disease.
Retinal cavernous hemangiomas are composed of clusters of angiomatous retinal vascular lesions. The racemose
aneurysm seen in Wyburn-Mason syndrome consists of a dilated arteriovenous communication in the retinal vasculature.
Choroidal hemangiomas involve anomalous choroidal, not retinal, vasculature
Question 122 of 130
What feature of peripheral degenerative retinoschisis, if present, increases the likelihood of this diagnosis?

Unilateral

Associated with an absolute scotoma

Symptomatic

Laser uptake at edge of, but not within schesis


Please select an answer
Feedback: Peripheral degenerative retinoschisis is associated with an absolute scotoma. Retinoschisis is asymptomatic
unless the absolute scotoma extends and is noticed posterior to the equator or if schesis-associated retinal detachment
develops. Retinoschisis is usually bilateral. Laser photocoagulation whitens the retina under a retinoshisis cavity.

Question 123 of 130


What exposure represents the most significant risk factor for the development of diabetic macular edema?

Nocturnal systemic hypotension

Oral statin therapy

Poor glycemic control

Hyperviscosity
Please select an answer
Feedback: Poor glycemic control is the greatest and most consistently demonstrated risk factor for the development of
diabetic macular edema. Oral statin therapy does not have a proven impact, but has been proposed to lower, rather than
increase the risk of progression of diabetic retinopathy. Hyperviscosity has been associated with venous occlusive disorders,
but not risk of diabetic macular edema. Diabetic macular edema is more common in patients with a longer duration of
diabetes mellitus and with uncontrolled systemic hypertension.

Question 124 of 130

What is the most common cause of vision loss in a patient with a choroidal osteoma?

Vitreous hemorrhage

Retinal detachment

Macular edema

Choroidal neovascularization
Please select an answer
Feedback: Choroidal neovascularization (CNV) is the most common cause of vision loss in patients with choroidal osteoma.
In the absence of CNV, retinal detachment, vitreous hemorrhage, and macular edema are uncommon causes of vision loss
in patients with choroidal osteoma.
Question 125 of 130
What is the most common cause of cystoid macular edema that demonstrates a fluorescein angiographic hyperfluorescent
cystic ("petaloid") pattern?

Retinitis pigmentosa

Niacin

Cataract surgery

Choroidal neovascularization
Please select an answer
Feedback: Of the causes listed, cataract surgery is the most common cause of clinical and angiographic cystoid macular
edema. Retinitis pigmentosa is a rare cause of CME. Although choroidal neovascularization may be associated with cystic
retinal thickening on optical coherence tomography, it rarely demonstrates a hyperfluorescent cystic pattern. Niacin toxicity
does not demonstrate retinal cystic hyperfluorescence.

Question 126 of 130


What is the most preferred management for acute traumatic choroidal rupture?

Intravitreal ranibizumab therapy

Pars plana vitrectomy

Observation

Laser photocoagulation
Please select an answer
Feedback: Observation is the preferred initial therapy for acute traumatic choroidal rupture. Intravitreal ranibizumab therapy
or laser photocoagulation may be indicated for choroidal neovascularization which usually presents months to years later.
Pars plana vitrectomy may be indicated for a nonclearing traumatic vitreous hemorrhage in an eye with a coexisting
choroidal rupture, but not for the choroidal rupture itself.

Question 127 of 130

Which of the following is the most-commonly administered therapy for cytomegaloviral retinitis?

Intravenous cidofovir

Intravitreal foscarnet

Intravitreal ganciclovir

Oral valganciclovir
Please select an answer
Feedback: Oral valganciclovir is currently the most-commonly administered therapy for cytomegaloviral retinitis. Less-
commonly used therapies include intravitreal ganciclovir, intravitreal foscarnet, and intravenous cidofovir.
Question 128 of 130
What predisposing factor contributes to an accelerated development of hydroxychloroquine maculopathy?

Northern European ancestry

Renal and/or hepatic disease

Obesity

Adolescence
Please select an answer
Feedback: The coexistence of renal and/or hepatic disease represents a risk factor for the development of
hydroxychloroquine maculopathy; youth, obesity, and Northern European ancestry do not.

Question 129 of 130


During the evaluation of retinal vasculitis, what test result would suggest a specific form of systemic vasculitis?

Sedimentation rate in polyarteritis nodosa.

Eosinophilia in Churg-Strauss syndrome.

p-ANCA in Wegener's granulomatosis.

anti-nuclear antibody in giant-cell arteritis.


Please select an answer
Feedback: Eosinophilia is found in Churg-Strauss vasculitis. Giant cell arteritis is associated with non-specific inflammatory
markers such as sedimentation rate and C-reactive protein. Similarl polyarteritis nodosa is not associated with specific test
result. The most specific test for Wegener's granulomatosis is c-ANCA, not p-ANCA.

Question 130 of 130


What is the most-commonly administered antimicrobial agent used in the management of delayed-onset, post-cataract
endophthalmitis?

Intravitreal vancomycin

Intravitreal amphotericin

Oral fluconazole

Intravitreal gentamicin
Please select an answer
Feedback: Intravitreal vancomycin is the most-commonly administered antimicrobial agent used in the management
of delayed-onset post-cataract endophthalmitis. Propionobacterium acnes, the most common cause of this entity, is
typically sensitive to vancomycin. Intravitreal gentamicin has the potential for retinal toxicity and is not commonly
used in the management of chronic endophthalmitis. Oral flucanazole and intravitreal amphotericin may be used in
the management of the less-common fungal delayed-onset post-cataract endophthalmitis.

Collected from www.aao.org (self assessment)


By Dr. AlBaraa AlQassimi

Das könnte Ihnen auch gefallen